Difference between revisions of "2016 AMC 12A Problems/Problem 3"

(Solution)
(Solution)
Line 6: Line 6:
 
<cmath>=\frac{3}{8}-\frac{2}{5}</cmath>
 
<cmath>=\frac{3}{8}-\frac{2}{5}</cmath>
 
<cmath>=\boxed{\textbf{(B)}-\frac{1}{40}}</cmath>
 
<cmath>=\boxed{\textbf{(B)}-\frac{1}{40}}</cmath>
Notice that it would not matter if either <math>-\frac{2}{5}</math> or <math>\frac{2}{5}</math> were used.
 

Revision as of 23:16, 3 February 2016

Solution

\[\text{rem}\left(\frac{3}{8},-\frac{2}{5}\right)\] \[=\frac{3}{8}-\left(-\frac{2}{5}\right)\lfloor\frac{\frac{3}{8}}{-\frac{-2}{5}}\rfloor\] \[=\frac{3}{8}+\left(\frac{2}{5}\right)\lfloor -\frac{15}{16}\rfloor\] \[=\frac{3}{8}+\left(\frac{2}{5}\right)\left(-1\right)\] \[=\frac{3}{8}-\frac{2}{5}\] \[=\boxed{\textbf{(B)}-\frac{1}{40}}\]